Bernstein Sets Are Baire Spaces

A topological space X is a Baire space if the intersection of any countable family of open and dense sets in X is dense in X (or equivalently, every nonempty open subset of X is of second category in X). One version of the Baire category theorem implies that every complete metric space is a Baire space. The real line \mathbb{R} with the usual Euclidean metric \lvert x-y \lvert is a complete metric space, and hence is a Baire space. The space of irrational numbers \mathbb{P} is also a complete metric space (not with the usual metric \lvert x-y \lvert but with another suitable metric that generates the Euclidean topology on \mathbb{P}) and hence is also a Baire space. In this post, we show that there are subsets of the real line that are Baire space but not complete metric spaces. These sets are called Bernstein sets.

A Bernstein set, as discussed here, is a subset B of the real line such that both B and \mathbb{R}-B intersect with every uncountable closed subset of the real line. We present an algorithm on how to generate such a set. Bernstein sets are not Lebesgue measurable. Our goal here is to show that Bernstein sets are Baire spaces but not weakly \alpha-favorable, and hence are spaces in which the Banach-Mazur game is undecidable.

Baire spaces are defined and discussed in this post. The Banach-Mazur game is discussed in this post. The algorithm of constructing Bernstein set is found in [2] (Theorem 5.3 in p. 23). Good references for basic terms are [1] and [3].
____________________________________________________________________________

In constructing Bernstein sets, we need the following lemmas.

Lemma 1
In the real line \mathbb{R}, any uncountable closed set has cardinality continuum.

Proof
In the real line, every uncountable subset of the real line has a limit point. In fact every uncountable subset of the real line contains at least one of its limit points (see The Lindelof property of the real line). Let A \subset \mathbb{R} be an uncountable closed set. The set A has to contain at least one of its limit point. As a result, at most countably many points of A are not limit points of A. Take away these countably many points of A that are not limit points of A and call the remainder A^*. The set A^* is still an uncountable closed set but with an additional property that every point of A^* is a limit point of A^*. Such a set is called a perfect set. In Perfect sets and Cantor sets, II, we demonstrate a procedure for constructing a Cantor set out of any nonempty perfect set. Thus A^* (and hence A) contains a Cantor set and has cardinality continuum. \blacksquare

Lemma 2
In the real line \mathbb{R}, there are continuum many uncountable closed subsets.

Proof
Let \mathcal{B} be the set of all open intervals with rational endpoints, which is a countable set. The set \mathcal{B} is a base for the usual topology on \mathbb{R}. Thus every nonempty open subset of the real line is the union of some subcollection of \mathcal{B}. So there are at most continuum many open sets in \mathbb{R}. Thus there are at most continuum many closed sets in \mathbb{R}. On the other hand, there are at least continuum many uncountable closed sets (e.g. [-b,b] for b \in \mathbb{R}). Thus we can say that there are exactly continuum many uncountable closed subsets of the real line. \blacksquare

Constructing Bernstein Sets

Let c denote the cardinality of the real line \mathbb{R}. By Lemma 2, there are only c many uncountable closed subsets of the real line. So we can well order all uncountable closed subsets of \mathbb{R} in a collection indexed by the ordinals less than c, say \left\{F_\alpha: \alpha < c \right\}. By Lemma 1, each F_\alpha has cardinality c. Well order the real line \mathbb{R}. Let \prec be this well ordering.

Based on the well ordering \prec, let x_0 and y_0 be the first two elements of F_0. Let x_1 and y_1 be the first two elements of F_1 (based on \prec) that are different from x_0 and y_0. Suppose that \alpha < c and that for each \beta < \alpha, points x_\beta and y_\beta have been selected. Then F_\alpha-\bigcup_{\beta<\alpha} \left\{x_\beta,y_\beta \right\} is nonempty since F_\alpha has cardinality c and only less than c many points have been selected. Then let x_\alpha and y_\alpha be the first two points of F_\alpha-\bigcup_{\beta<\alpha} \left\{x_\beta,y_\beta \right\} (according to \prec). Thus x_\alpha and y_\alpha can be chosen for each \alpha<c.

Let B=\left\{ x_\alpha: \alpha<c \right\}. Then B is a Bernstein set. Note that B meets every uncountable closed set F_\alpha with the point x_\alpha and the complement of B meets every uncountable closed set F_\alpha with the point y_\alpha.

The algorithm described here produces a unique Bernstein set that depends on the ordering of the uncountable closed sets F_\alpha and the well ordering \prec of \mathbb{R}.

____________________________________________________________________________

Key Lemmas

Baire spaces are defined and discussed in this previous post. Baire spaces can also be characterized using the Banach-Mazur game. The following lemmas establish that any Bernstein is a Baire space that is not weakly \alpha-favorable. Lemma 3 is applicable to all topological spaces. Lemmas 4, 5, 6, and 7 are specific to the real line.

Lemma 3
Let Y be a topological space. Let F \subset Y be a set of first category in Y. Then Y-F contains a dense G_\delta subset.

Proof
Let F \subset Y be a set of first category in Y. Then F=\bigcup \limits_{n=0}^\infty F_n where each F_n is nowhere dense in Y. The set X-\bigcup \limits_{n=0}^\infty \overline{F_n} is a dense G_\delta set in the space X and it is contained in the complement of F. We have:

\displaystyle . \ \ \ \ \ X-\bigcup \limits_{n=0}^\infty \overline{F_n} \subset X-F \blacksquare

We now set up some notaions in preparation of proving Lemma 4 and Lemma 7. For any set A \subset \mathbb{R}, let \text{int}(A) be the interior of the set A. Denote each positive integer n by n=\left\{0,1,\cdots,n-1 \right\}. In particular, 2=\left\{0,1\right\}. Let 2^{n} denote the collection of all functions f: n \rightarrow 2. Identify each f \in 2^n by the sequence f(0),f(1),\cdots,f(n-1). This identification makes notations in the proofs of Lemma 4 and Lemma 7 easier to follow. For example, for f \in 2^n, I_f denotes a closed interval I_{f(0),f(1),\cdots,f(n-1)}. When we choose two disjoint subintervals of this interval, they are denoted by I_{f,0} and I_{f,1}. For f \in 2^n, f \upharpoonright 1 refers to f(0), f \upharpoonright 2 refers to the sequence f(0),f(1), and f \upharpoonright 3 refers to the sequence f(0),f(1),f(2) and so on.

The Greek letter \omega denotes the first infinite ordinal. We equate it as the set of all nonnegative integers \left\{0,1,2,\cdots \right\}. Let 2^\omega denote the set of all functions from \omega to 2=\left\{0,1 \right\}.

Lemma 4
Let W \subset \mathbb{R} be a dense G_\delta set. Let U be a nonempty open subset of \mathbb{R}. Then W \cap U contains a Cantor set (hence an uncountable closed subset of the real line).

Proof
Let W=\bigcap \limits_{n=0}^\infty O_n where each O_n is an open and dense subset of \mathbb{R}. We describe how a Cantor set can be obtained from the open sets O_n. Take a closed interval I_\varnothing=[a,b] \subset O_0 \cap U. Let C_0=I_\varnothing. Then pick two disjoint closed intervals I_{0} \subset O_1 and I_{1} \subset O_1 such that they are subsets of the interior of I_\varnothing and such that the lengths of both intervals are less than 2^{-1}. Let C_1=I_0 \cup I_1.

At the n^{th} step, suppose that all closed intervals I_{f(0),f(1),\cdots,f(n-1)} (for all f \in 2^n) are chosen. For each such interval, we pick two disjoint closed intervals I_{f,0}=I_{f(0),f(1),\cdots,f(n-1),0} and I_{f,1}=I_{f(0),f(1),\cdots,f(n-1),1} such that each one is subset of O_n and each one is subset of the interior of the previous closed interval I_{f(0),f(1),\cdots,f(n-1)} and such that the lenght of each one is less than 2^{-n}. Let C_n be the union of I_{f,0} \cup I_{f,1} over all f \in 2^n.

Then C=\bigcap \limits_{j=0}^\infty C_j is a Cantor set that is contained in W \cap U. \blacksquare

Lemma 5
Let X \subset \mathbb{R}. If X is not of second category in \mathbb{R}, then \mathbb{R}-X contains an uncountable closed subset of \mathbb{R}.

Proof
Suppose X is of first category in \mathbb{R}. By Lemma 3, the complement of X contains a dense G_\delta subset. By Lemma 4, the complement contains a Cantor set (hence an uncountable closed set). \blacksquare

Lemma 6
Let X \subset \mathbb{R}. If X is not a Baire space, then \mathbb{R}-X contains an uncountable closed subset of \mathbb{R}.

Proof
Suppose X \subset \mathbb{R} is not a Baire space. Then there exists some open set U \subset X such that U is of first category in X. Let U^* be an open subset of \mathbb{R} such that U^* \cap X=U. We have U=\bigcup \limits_{n=0}^\infty F_n where each F_n is nowhere dense in X. It follows that each F_n is nowhere dense in \mathbb{R} too.

By Lemma 3, \mathbb{R}-U contains W, a dense G_\delta subset of \mathbb{R}. By Lemma 4, there is a Cantor set C contained in W \cap U^*. This uncountable closed set C is contained in \mathbb{R}-X. \blacksquare

Lemma 7
Let X \subset \mathbb{R}. Suppose that X is a weakly \alpha-favorable space. If X is dense in the open interval (a,b), then there is an uncountable closed subset C of \mathbb{R} such that C \subset X \cap (a,b).

Proof
Suppose X is a weakly \alpha-favorable space. Let \gamma be a winning strategy for player \alpha in the Banach-Mazur game BM(X,\beta). Let (a,b) be an open interval in which X is dense. We show that a Cantor set can be found inside X \cap (a,b) by using the winning strategy \gamma.

Let I_{-1}=[a,b]. Let t=b-a. Let U_{-1}^*=(a,b) and U_{-1}=U^* \cap X. We take U_{-1} as the first move by the player \beta. Then the response made by \alpha is V_{-1}=\gamma(U_{-1}). Let C_{-1}=I_{-1}.

Choose two disjoint closed intervals I_0 and I_1 that are subsets of the interior of I_{-1} such that the lengths of these two intervals are less than 2^{-t} and such that U_0^*=\text{int}(I_0) and U_1^*=\text{int}(I_1) satisfy further properties, which are that U_0=U_0^* \cap X \subset V_{-1} and U_1=U_1^* \cap X \subset V_{-1} are open in X. Let U_0 and U_1 be two possible moves by player \beta at the next stage. Then the two possible responses by \alpha are V_0=\gamma(U_{-1},U_0) and V_1=\gamma(U_{-1},U_1). Let C_1=I_0 \cup I_1.

At the n^{th} step, suppose that for each f \in 2^n, disjoint closed interval I_f=I_{f(0),\cdots,f(n-1)} have been chosen. Then for each f \in 2^n, we choose two disjoint closed intervals I_{f,0} and I_{f,1}, both subsets of the interior of I_f, such that the lengths are less than 2^{-(n+1) t}, and:

  • U_{f,0}^*=\text{int}(I_{f,0}) and U_{f,1}^*=\text{int}(I_{f,1}),
  • U_{f,0}=U_{f,0}^* \cap X and U_{f,1}=U_{f,1}^* \cap X are open in X,
  • U_{f,0} \subset V_f and U_{f,1} \subset V_f

We take U_{f,0} and U_{f,1} as two possible new moves by player \beta from the path f \in 2^n. Then let the following be the responses by player \alpha:

  • V_{f,0}=\gamma(U_{-1},U_{f \upharpoonright 1}, U_{f \upharpoonright 2}, \cdots,U_{f \upharpoonright (n-1)},U_f, U_{f,0})
  • V_{f,1}=\gamma(U_{-1},U_{f \upharpoonright 1}, U_{f \upharpoonright 2}, \cdots,U_{f \upharpoonright (n-1)},U_f, U_{f,1})

The remaining task in the n^{th} induction step is to set C_n=\bigcup \limits_{f \in 2^n} I_{f,0} \cup I_{f,1}.

Let C=\bigcap \limits_{n=-1}^\infty C_n, which is a Cantor set, hence an uncountable subset of the real line. We claim that C \subset X.

Let x \in C. There there is some g \in 2^\omega such that \left\{ x \right\} = \bigcap \limits_{n=1}^\infty I_{g \upharpoonright n}. The closed intervals I_{g \upharpoonright n} are associated with a play of the Banach-Mazur game on X. Let the following sequence denote this play:

\displaystyle (1) \ \ \ \ \ U_{-1},V_{-1},U_{g \upharpoonright 1},V_{g \upharpoonright 1},U_{g \upharpoonright 2},V_{g \upharpoonright 2},U_{g \upharpoonright 3},U_{g \upharpoonright 3}, \cdots

Since the strategy \gamma is a winning strategy for player \alpha, the intersection of the open sets in (1) must be nonempty. Thus \bigcap \limits_{n=1}^\infty V_{g \upharpoonright n} \ne \varnothing.

Since the sets V_{g \upharpoonright n} \subset I_{g \upharpoonright n}, and since the lengths of I_{g \upharpoonright n} go to zero, the intersection must have only one point, i.e., \bigcap \limits_{n=1}^\infty V_{g \upharpoonright n} = \left\{ y \right\} for some y \in X. It also follows that y=x. Thus x \in X. We just completes the proof that X contains an uncountable closed subset of the real line. \blacksquare

____________________________________________________________________________

Conclusions about Bernstein Sets

Lemma 6 above establishes that any Bernstein set is a Baire space (if it isn’t, the complement would contain an uncountable closed set). Lemma 7 establishes that any Bernstein set is a topological space in which the player \alpha has no winning strategy in the Banach-Mazur game (if player \alpha always wins in a Bernstein set, it would contain an uncountable closed set). Thus any Bernstein set cannot be a weakly \alpha favorable space. According to this previous post about the Banach-Mazur game, Baire spaces are characterized as the spaces in which the player \beta has no winning strategy in the Banach-Mazur game. Thus any Bernstein set in a topological space in which the Banach-Mazur game is undecidable (i.e. both players in the Banach-Mazur game have no winning strategy).

One interesting observation about Lemma 6 and Lemma 7. Lemma 6 (as well as Lemma 5) indicates that the complement of a “thin” set contains a Cantor set. On the other hand, Lemma 7 indicates that a “thick” set contains a Cantor set (if it is dense in some open interval).

Reference

  1. Engelking, R., General Topology, Revised and Completed edition, Heldermann Verlag, Berlin, 1989.
  2. Oxtoby, J. C., Measure and Category, Graduate Texts in Mathematics, Springer-Verlag, New York, 1971.
  3. Willard, S., General Topology, Addison-Wesley Publishing Company, 1970.

The Banach-Mazur Game

A topological space X is said to be a Baire space if for every countable family \left\{U_0,U_1,U_2,\cdots \right\} of open and dense subsets of X, the intersection \bigcap \limits_{n=0}^\infty U_n is dense in X (equivalently if every nonempty open subset of X is of second category in X). By the Baire category theorem, every complete metric space is a Baire space. The Baire property (i.e. being a Baire space) can be characterized using the Banach-Mazur game, which is the focus of this post.

Baire category theorem and Baire spaces are discussed in this previous post. We define the Banach-Mazur game and show how this game is related to the Baire property. We also define some completeness properties stronger than the Baire property using this game. For a survey on Baire spaces, see [4]. For more information about the Banach-Mazur game, see [1]. Good references for basic topological terms are [3] and [5]. All topological spaces are assumed to be at least Hausdorff.

__________________________________________________________________________

The Banach-Mazur Game

The Banach-Mazur game is a two-person game played on a topological space. Let X be a space. There are two players, \alpha and \beta. They take turn choosing nested decreasing nonempty open subsets of X as follows. The player \beta goes first by choosing a nonempty open subset U_0 of X. The player \alpha then chooses a nonempty open subset V_0 \subset U_0. At the nth play where n \ge 1, \beta chooses an open set U_n \subset V_{n-1} and \alpha chooses an open set V_n \subset U_n. The player \alpha wins if \bigcap \limits_{n=0}^\infty V_n \ne \varnothing. Otherwise the player \beta wins.

If the players in the game described above make the moves U_0,V_0,U_1,V_1,U_2,V_2,\cdots, then this sequence of open sets is said to be a play of the game.

The Banach-Mazur game, as described above, is denoted by BM(X,\beta). In this game, the player \beta makes the first move. If we modify the game by letting \alpha making the first move, we denote this new game by BM(X,\alpha). In either version, the goal of player \beta is to reach an empty intersection of the chosen open sets while player \alpha wants the chosen open sets to have nonempty intersection.

A Remark About Topological Games

Before relating the Banach-Mazur game to Baire spaces, we give a remark about topological games. For any two-person game played on a topological space, we are interested in the following question.

  • Can a player, by making his/her moves judiciously, insure that he/she will always win no matter what moves the other player makes?

If the answer to this question is yes, then the player in question is said to have a winning strategy. For an illustration, consider a space X that is of first category in itself, so that X=\bigcup \limits_{n=0}^\infty X_n where each X_n is nowhere dense in X. Then player \beta has a winning strategy in the Banach-Mazur game BM(X,\beta). The player \beta always wins the game by making his/her nth play U_n \subset V_{n-1} - \overline{X_n}.

In general, a strategy for a player in a game is a rule that specifies what moves he/she will make in every possible situation. In other words, a strategy for a player is a function whose domain is the set of all partial plays of the game, and this function tells the player what the next move should be. A winning strategy for a player is a strategy such that this player always wins if that player makes his/her moves using this strategy. A strategy for a player in a game is not a winning strategy if of all the plays of the game resulting from using this strategy, there is at least one specific play of the game resulting in a win for the other player.

__________________________________________________________________________

Strategies in the Banach-Mazur Game

With the above discussion in mind, let us discuss the strategies in the Banach-Mazur game. We show that the strategies in this game code a great amount of information about the topological space in which the game is played.

First we discuss strategies for player \beta in the game BM(X,\beta). A strategy for player \beta is a function \sigma such that U_0=\sigma(\varnothing) (the first move) and for each partial play of the game (n \ge 1)

\displaystyle (*) \ \ \ \ \ \ U_0,V_0,U_1,V_1,\cdots,U_{n-1},V_{n-1},

U_n=\sigma(U_0,V_0,U_1,V_1,\cdots,U_{n-1},V_{n-1}) is a nonempty open set such that U_n \subset V_{n-1}. If player \beta makes all his/her moves using the strategy \sigma, then the strategy \sigma for player \beta contains information on all moves of \beta. We adopt the convention that a strategy for a player in a game depends only on the moves of the other player. Thus for the partial play of the Banach-Mazur game denoted by (*) above, U_n=\sigma(V_0,V_1,\cdots,V_{n-1}).

If \sigma is a winning strategy for player \beta in the game BM(X,\beta), then using this strategy will always result in a win for \beta. On the other hand, if \sigma is a not a winning strategy for player \beta in the game BM(X,\beta), then there exists a specific play of the Banach-Mazur game

\displaystyle . \ \ \ \ \ \ U_0,V_0,U_1,V_1,\cdots,U_{n-1},V_{n-1},\cdots

such that U_0=\sigma(\varnothing), and for each n \ge 1, U_n=\sigma(V_0,\cdots,V_{n-1}) and player \alpha wins in this play of the game, that is, \bigcap \limits_{n=0}^\infty V_n \ne \varnothing.

In the game BM(X,\alpha) (player \alpha making the first move), a strategy for player \beta is a function \gamma such that for each partial play of the game

\displaystyle (**) \ \ \ \ \ V_0,U_1,V_1,\cdots,U_{n-1},V_{n-1},

U_n=\gamma(V_0,V_1,\cdots,V_{n-1}) is a nonempty open set such that U_n \subset V_{n-1}.

We now present a lemma that helps translate game information into topological information.

Lemma 1
Let X be a space. Let O \subset X be a nonempty open set. Let \tau be the set of all nonempty open subsets of O. Let f: \tau \longrightarrow \tau be a function such that for each V \in \tau, f(V) \subset V. Then there exists a disjoint collection \mathcal{U} consisting of elements of f(\tau) such that \bigcup \mathcal{U} is dense in O.

Proof
This is an argument using Zorn’s lemma. If the open set O in the hypothesis has only one point, then the conclusion of the lemma holds. So assume that O has at least two points.

Let \mathcal{P} be the set consisting of all collections \mathcal{F} such that each \mathcal{F} is a disjoint collection consisting of elements of f(\tau). First \mathcal{P} \ne \varnothing. To see this, let V and W be two disjoint open sets such that V \subset O and W \subset O. This is possible since O has at least two points. Let \mathcal{F^*}=\left\{ f(V),f(W)\right\}. Then we have \mathcal{F^*} \in \mathcal{P}. Order \mathcal{P} by set inclusion. It is straightforward to show that (\mathcal{P}, \subset) is a partially ordered set.

Let \mathcal{T} \subset \mathcal{P} be a chain (a totally ordered set). We wish to show that \mathcal{T} has an upper bound in \mathcal{P}. The candidate for an upper bound is \bigcup \mathcal{T} since it is clear that for each \mathcal{F} \in \mathcal{T}, \mathcal{F} \subset \bigcup \mathcal{T}. We only need to show \bigcup \mathcal{T} \in \mathcal{P}. To this end, we need to show that any two elements of \bigcup \mathcal{T} are disjoint open sets.

Note that elements of \bigcup \mathcal{T} are elements of f(\tau). Let T_1,T_2 \in \bigcup \mathcal{T}. Then T_1 \in \mathcal{F}_1 and T_2 \in \mathcal{F}_2 for some \mathcal{F}_1 \in \mathcal{T} and \mathcal{F}_2 \in \mathcal{T}. Since \mathcal{T} is a chain, either \mathcal{F}_1 \subset \mathcal{F}_2 or \mathcal{F}_2 \subset \mathcal{F}_1. This means that T_1 and T_2 belong to the same disjoing collection in \mathcal{T}. So they are disjoint open sets that are members of f(\tau).

By Zorn’s lemma, (\mathcal{P}, \subset) has a maximal element \mathcal{U}, which is a desired disjoint collection of sets in f(\tau). Since \mathcal{U} is maximal with respect to \subset, \bigcup \mathcal{U} is dense in O. \blacksquare

__________________________________________________________________________

Characterizing Baire Spaces using the Banach-Mazur Game

Lemma 1 is the linkage between the Baire property and the strategies in the Banach-Mazur game. The thickness in Baire spaces and spaces of second category allow us to extract a losing play in any strategy for player \beta. The proofs for both Theorem 1 and Theorem 2 are very similar (after adjusting for differences in who makes the first move). Thus we only present the proof for Theorem 1.

Theorem 1
The space X is a Baire space if and only if player \beta has no winning strategy in the game BM(X,\beta).

Proof
\Longleftarrow Suppose that X is not a Baire space. We define a winning strategy in the game BM(X,\beta) for player \beta. The space X not being a Baire space implies that there is some nonempty open set U_0 \subset X such that U_0 is of first category in X. Thus U_0=\bigcup \limits_{n=1}^\infty F_n where each F_n is nowhere dense in X.

We now define a winning strategy for \beta. Let U_0 be the first move of \beta. For each n \ge 1, let player \beta make his/her move by letting U_n \subset V_{n-1} - \overline{F_n} if V_{n-1} is the last move by \alpha. It is clear that whenever \beta chooses his/her moves in this way, the intersection of the open sets has to be empty.

\Longrightarrow Suppose that X is a Baire space. Let \sigma be a strategy for the player \beta. We show that \sigma cannot be a winning strategy for \beta.

Let U_0=\sigma(\varnothing) be the first move for \beta. For each open V_0 \subset U_0, \sigma(V_0) \subset V_0. Apply Lemma 1 to obtain a disjoint collection \mathcal{U}_0 consisting of open sets of the form \sigma(V_0) such that \bigcup \mathcal{U}_0 is dense in U_0.

For each W=\sigma(V_0) \in \mathcal{U}_0, we have \sigma(V_0,V_1) \subset V_1 for all open V_1 \subset W. So the function \sigma(V_0,\cdot) is like the function f in Lemma 1. We can then apply Lemma 1 to obtain a disjoint collection \mathcal{U}_1(W) consisting of open sets of the form \sigma(V_0,V_1) such that \bigcup \mathcal{U}_1(W) is dense in W. Then let \mathcal{U}_1=\bigcup_{W \in \mathcal{U}_0} \mathcal{U}_1(W). Based on how \mathcal{U}_1(W) are obtained, it follows that \bigcup \mathcal{U}_1 is dense in U_0.

Continue the inductive process in the same manner, we can obtain, for each n \ge 1, a disjoint collection \mathcal{U}_n consisting of open sets of the form \sigma(V_0,\dots,V_{n-1}) (these are moves made by \beta using the strategy \sigma) such that \bigcup \mathcal{U}_n is dense in U_0.

For each n, let O_n=\bigcup \mathcal{U}_n. Each O_n is dense open in U_0. Since X is a Baire space, every nonempty open subset of X is of second category in X (including U_0). Thus \bigcap \limits_{n=0}^\infty O_n \ne \varnothing. From this nonempty intersection, we can extract a play of the game such that the open sets in this play of the game have one point in common (i.e. player \alpha wins). We can extract the play of the game because the collection \mathcal{U}_n are disjoint. Thus the strategy \sigma is not a winning strategy for \beta. This completes the proof of Theorem 1. \blacksquare

Theorem 2
The space X is of second category in itself if and only if player \beta has no winning strategy in the game BM(X,\alpha).

__________________________________________________________________________

Some Completeness Properties

Theorem 1 shows that a Baire space is one in which the player \beta has no winning strategy in the Banach-Mazur game (the version in which \beta makes the first move). In such a space, no matter what strategy player \beta wants to use, it can be foiled by player \alpha by producing one specific play in which \beta loses. We now consider spaces in which player \alpha has a winning strategy. A space X is said to be a weakly \alpha-favorable if player \alpha has a winning strategy in the game BM(X,\beta). If \alpha always wins, then \beta has no winning strategy. Thus the property of being a weakly \alpha-favorable space is stronger than the Baire property.

In any complete metric space, the player \alpha always has a winning strategy. The same idea used in proving the Baire category theorem can be used to establish this fact. By playing the game in a complete metric space, player \alpha can ensure a win by making sure that the closure of his/her moves have diameters converge to zero (and the closure of his/her moves are subsets of the previous moves).

Based on Theorem 1, any Baire space is a space in which player \beta of the Banach-Mazur game has no winning strategy. Any Baire space that is not weakly \alpha-favorable is a space in which both players of the Banach-Mazur game have no winning strategy (i.e. the game is undecidable). Any subset of the real line \mathbb{R} that is a Bernstein set is such a space. A subset B of the real line is said to be a Bernstein set if B and its complement intersect every uncountable closed subset of the real line. Bernstein sets are discussed here.

Suppose \theta is a strategy for \alpha in the game BM(X,\beta). If at each step, the strategy \theta can provide a move based only on the other player’s last move, it is said to be a stationary strategy. For example, in the partial play U_0,V_0,\cdots,U_{n-1},V_{n-1},U_n, the strategy \theta can determine the next move for \alpha by only knowing the last move of \beta, i.e., V_n=\theta(U_n). A space X is said to be \alpha-favorable if player \alpha has a stationary winning strategy in the game BM(X,\beta). Clearly, any \alpha-favorable spaces are weakly \alpha-favorable spaces. However, there are spaces in which player \alpha has a winning strategy in the Banach-Mazur game and yet has no stationary winning strategy (see [2]). Stationary winning strategy for \alpha is also called \alpha-winning tactic (see [1]).

Reference

  1. Choquet, G., Lectures on analysis, Vol I, Benjamin, New York and Amsterdam, 1969.
  2. Deb, G., Stategies gagnantes dans certains jeux topologiques, Fund. Math. 126 (1985), 93-105.
  3. Engelking, R., General Topology, Revised and Completed edition, 1989, Heldermann Verlag, Berlin.
  4. Haworth, R. C., McCoy, R. A., Baire Spaces, Dissertations Math., 141 (1977), 1 – 73.
  5. Willard, S., General Topology, 1970, Addison-Wesley Publishing Company.

____________________________________________________________________

Revised 4/4/2014. \copyright \ 2014 \text{ by Dan Ma}

A Question About The Rational Numbers

Let \mathbb{R} be the real line and \mathbb{Q} be the set of all rational numbers. Consider the following question:

___________________________________________________________________________________
Question

  • For each nonnegative integer n, let U_n be an open subset of \mathbb{R} such that that \mathbb{Q} \subset U_n. The intersection \bigcap \limits_{n=0}^\infty U_n is certainly nonempty since it contains \mathbb{Q}. Does this intersection necessarily contain some irrational numbers?

___________________________________________________________________________________

While taking a real analysis course, the above question was posted to the author of this blog by the professor. Indeed, the question is an excellent opening of the subject of category. We first discuss the Baire category theorem and then discuss the above question. A discussion of Baire spaces follow. For any notions not defined here and for detailed discussion of any terms discussed here, see [1] and [2].

In the above question, the set \bigcap \limits_{n=0}^\infty U_n is a G_\delta set since it is the intersection of countably many open sets. It is also dense in the real line \mathbb{R} since it contains the rational numbers. So the question can be rephrased as: is the set of rational numbers \mathbb{Q} a G_\delta set? Can a dense G_\delta set in the real line \mathbb{R} be a “small” set such as \mathbb{Q}? The discussion below shows that \mathbb{Q} is too “thin” to be a dense G_\delta set. Put it another way, a dense G_\delta subset of the real line is a “thick” set. First we present the Baire category theorem.
___________________________________________________________________________________

Baire Category Theorem

Let X be a complete metric space. For each nonnegative integer n, let O_n be an open subset of X that is also dense in X. Then \bigcap \limits_{n=0}^\infty O_n is dense in X.

Proof
Let A=\bigcap \limits_{n=0}^\infty O_n. Let V_0 be any nonempty open subset of X. We show that V_0 contains some point of A.

Since O_0 is dense in X, V_0 contains some point of O_0. Let x_0 be one such point and choose open set V_1 such that x_0 \in V_1 and \overline{V_1} \subset V_0 \cap O_0 \subset V_0 with the additional condition that the diameter of \overline{V_1} is less than \displaystyle \frac{1}{2^1}.

Since O_1 is dense in X, V_1 contains some point of O_1. Let x_1 be one such point and choose open set V_2 such that x_1 \in V_2 and \overline{V_2} \subset V_1 \cap O_1 \subset V_1 with the additional condition that the diameter of \overline{V_2} is less than \displaystyle \frac{1}{2^2}.

By continuing this inductive process, we obtain a nested sequence of open sets V_n and a sequence of points x_n such that x_n \in V_n \subset \overline{V_n} \subset V_{n-1} \cap O_{n-1} \subset V_0 for each n and that the diameters of \overline{V_n} converge to zero (according to some complete metric on X). Then the sequence of points x_n is a Cauchy sequence. Since X is a complete metric space, the sequence x_n converges to a point x \in X.

We claim that x \in V_0 \cap A. To see this, note that for each n, x_j \in \overline{V_n} for each j \ge n. Since x is the sequential limit of x_j, x \in \overline{V_n} for each n. It follows that x \in O_n for each n (x \in A) and x \in V_0. This completes the proof of Baire category theorem. \blacksquare

___________________________________________________________________________________

Discussion of the Above Question

For each nonnegative integer n, let U_n be an open subset of \mathbb{R} such that that \mathbb{Q} \subset U_n. We claim that the intersection \bigcap \limits_{n=0}^\infty U_n contain some irrational numbers.

Suppose the intersection contains no irrational numbers, that is, \mathbb{Q}=\bigcap \limits_{n=0}^\infty U_n.

Let \mathbb{Q} be enumerated by \left\{r_0,r_1,r_2,\cdots \right\}. For each n, let G_n=\mathbb{R}-\left\{ r_n \right\}. Then each G_n is an open and dense set in \mathbb{R}. Note that the set of irrational numbers \mathbb{P}=\bigcap \limits_{n=0}^\infty G_n.

We then have countably many open and dense sets U_0,U_1,U_2,\cdots,G_0,G_1,G_2,\cdots whose intersection is empty. Note that any point that belongs to all U_n has to be a rational number and any point that belongs to all G_n has to be an irrational number. On the other hand, the real line \mathbb{R} with the usual metric is a complete metric space. By the Baire category theorem, the intersection of all U_n and G_n must be nonempty. Thus the intersection \bigcap \limits_{n=0}^\infty U_n must contain more than rational numbers.

It follows that the set of rational numbers \mathbb{Q} cannot be a G_\delta set in \mathbb{R}. In fact, the discussion below will show that the in a complete metric space such as the real line, any dense G_\delta set must be a “thick” set (see Theorem 3 below).
___________________________________________________________________________________

Baire Spaces

The version of the Baire category theorem discussed above involves complete metric spaces. However, the ideas behind the Baire category theorem are topological in nature. The following is the conclusion of the Baire category theorem:

(*) \ \ \ \ X is a topological space such that for each countable family \left\{U_0,U_1,U_2,\cdots \right\} of open and dense sets in X, the intersection \bigcap \limits_{n=0}^\infty U_n is dense in X.

A Baire space is a topological space in which the condition (*) holds. The Baire category theorem as stated above gives a sufficient condition for a topological space to be a Baire space. There are plenty of Baire spaces that are not complete metric spaces, in fact, not even metric spaces. The condition (*) is a topological property. In order to delve deeper into this property, let’s look at some related notions.

Let X be a topological space. A set A \subset X is dense in X if every open subset of X contains a point of A (i.e. \overline{A}=X). A set A \subset X is nowhere dense in X if for every open subset U of X, there is some open set V \subset U such that V contains no point of A (another way to describe this: \overline{A} contains no interior point of X).

A set is dense if its points can be found in every nonempty open set. A set is nowhere dense if every nonempty open set has an open subset that misses it. For example, the set of integers \mathbb{N} is nowhere dense in \mathbb{R}.

A set A \subset X is of first category in X if A is the union of countably many nowhere dense sets in X. A set A \subset X is of second category in X if it is not of first category in X.

To make sense of these notions, the following observation is key:

(**) \ \ \ \ F \subset X is nowhere dense in X if and only if X-\overline{F} is an open and dense set in X.

So in a Baire space, if you take away any countably many closed and nowhere dense sets (in other words, taking away a set of first category in X), there is a remainder (there are still points remaining) and the remainder is still dense in X. In thinking of sets of first category as “thin”, a Baire space is one that is considered “thick” or “fat” in that taking away a “thin” set still leaves a dense set.

A space X is of second category in X means that if you take away any countably many closed and nowhere dense sets in X, there are always points remaining. For a set Y \subset X, Y is of second category in X means that if you take away from Y any countably many closed and nowhere dense sets in X, there are still points remaining in Y. A set of second category is “thick” in the sense that after taking away a “thin” set there are still points remaining.

For example, \mathbb{N} is nowhere dense in \mathbb{R} and thus of first category in \mathbb{R}. However, \mathbb{N} is of second category in itself. In fact, \mathbb{N} is a Baire space since it is a complete metric space (with the usual metric).

For example, \mathbb{Q} is of first category in \mathbb{R} since it is the union of countably many singleton sets (\mathbb{Q} is also of first category in itself).

For example, let T=[0,1] \cup (\mathbb{Q} \cap [2,3]). The space T is not a Baire space since after taking away the rational numbers in [2,3], the remainder is no longer dense in T. However, T is of second category in itself.

For example, any Cantor set defined in the real line is nowhere dense in \mathbb{R}. However, any Cantor set is of second category in itself (in fact a Baire space).

The following theorems summarize these concepts.

Theorem 1a
Let X be a topological space. The following conditions are equivalent:

  1. X is of second category in itself.
  2. The intersection of countably many dense open sets is nonempty.

Theorem 1b
Let X be a topological space. Let A \subset X. The following conditions are equivalent:

  1. The set A is of second category in X.
  2. The intersection of countably many dense open sets in X must intersect A.

Theorem 2
Let X be a topological space. The following conditions are equivalent:

  1. X is a Baire space, i.e., the intersection of countably many dense open sets is dense in X.
  2. Every nonempty open subset of X is of second category in X.

The above theorems can be verified by appealing to the relevant definitions, especially the observation (**). Theorems 2 and 1a indicate that any Baire space is of second category in itself. The converse is not true (see the space T=[0,1] \cup (\mathbb{Q} \cap [2,3]) discussed above).

___________________________________________________________________________________

Dense G delta Subsets of a Baire Space

In answering the question stated at the beginning, we have shown that \mathbb{Q} cannot be a G_\delta set. Being a set of first category, \mathbb{Q} cannot be a dense G_\delta set. In fact, it can be shown that in a Baire space, any dense G_\delta subset is also a Baire space.

Theorem 3
Let X be a Baire space. Then any dense G_\delta subset of X is also a Baire space.

Proof
Let Y=\bigcap \limits_{n=0}^\infty U_n where each U_n is open and dense in X. We show that Y is a Baire space. In light of Theorem 2, we show that every nonempty open set of Y is of second category in Y.

Suppose that there is a nonempty open subset U \subset Y such that U is of first category in Y. Then U=\bigcup \limits_{n=0}^\infty W_n where each W_n is nowhere dense in Y. It can be shown that each W_n is also nowhere dense in X.

Since U is open in Y, there is an open set U^* \subset X such that U^* \cap Y=U. Note that for each n, F_n=X-U_n is closed and nowhere dense in X. Then we have:

\displaystyle (1) \ \ \ \ \ U^*=\bigcup \limits_{n=0}^\infty (F_n \cap U^*) \cup \bigcup \limits_{n=0}^\infty W_n

(1) shows that U^* is the union of countably many nowhere dense sets in X, contracting that every nonempty open subset of X is of second category in X. Thus we can conclude that every nonempty open subset of Y is of second category in Y. \blacksquare

Reference

  1. Engelking, R., General Topology, Revised and Completed edition, 1989, Heldermann Verlag, Berlin.
  2. Willard, S., General Topology, 1970, Addison-Wesley Publishing Company.

Revised July 3, 2019